LSAT and Law School Admissions Forum

Get expert LSAT preparation and law school admissions advice from PowerScore Test Preparation.

User avatar
 Dave Killoran
PowerScore Staff
  • PowerScore Staff
  • Posts: 5848
  • Joined: Mar 25, 2011
|
#79629
Complete Question Explanation
(The complete setup for this game can be found here: lsat/viewtopic.php?t=6157)

The correct answer choice is (B).

From our discussion of the connection possibilities, if K is O, then L is connected to M:

PT6-Oct 1992 LG Explanations game 4 #22 diagram 1.png
Answer choice (A) is false, and so it can be eliminated.

O is now closed to further connections, eliminating answer choices (C) and (E).

Answer choice (D) can be eliminated because L is connected to M, but it cannot connect to K or O, leaving only J as a possible connection. Thus, L can connect to at most two other islands, not three.

Answer choice (B) is the correct answer because J could connect to L or to M.
 mcduffeeee
  • Posts: 1
  • Joined: Mar 22, 2019
|
#63588
Hello,

I missed #22 of the Island Bridge game because I had to guess between answer choices B, C, and D. I eliminated answer choices A and E from my diagram of this scenario. My question is how to know to pick B? In my diagram B, C, and D could all be true. Here's my diagram for reference:

  • J -- L -- K
    |     /   \    |
    M ------ O
Thus, based on my diagramming, B) J connecting with L; C) L connecting with O; D) Three bridges connecting L with other islands could all occur. What am I doing wrong?

Thank-you!
User avatar
 Dave Killoran
PowerScore Staff
  • PowerScore Staff
  • Posts: 5848
  • Joined: Mar 25, 2011
|
#63590
mcduffeeee wrote:Hello,

I missed #22 of the Island Bridge game because I had to guess between answer choices B, C, and D. I eliminated answer choices A and E from my diagram of this scenario. My question is how to know to pick B? In my diagram B, C, and D could all be true. Here's my diagram for reference:

  • J -- L -- K
    |     /   \    |
    M ------ O
Thus, based on my diagramming, B) J connecting with L; C) L connecting with O; D) Three bridges connecting L with other islands could all occur. What am I doing wrong?

Thank-you!
Hi mcduffeeee,

Thanks for the question! First, in your diagram, you've violated the rule about "K is directly connected by bridge with exactly one island." Second, you are missing the last rule that "A bridge directly connects J with O." So, your diagram for this won't work :(

what we know is that when K connects to O, K then satisfies the sixth rule and is "closed." O also then has three connections (J, K, and M), so it too is closed. Via the fourth rule, that forces L to connect to M. Let's look at the answers:

Answer choice (A): This cannot be the case as L must connect to M.

Answer choice (B): This is the correct answer. J's second connection can be to either M or L.

Answer choice (C): Since O already has three connections (J, K, and M), L cannot connect to O.

Answer choice (D): L must connect to M, and it could connect to J. However, K and O are both closed, so the maximum number of islands L can connect to is two.

Answer choice (E): This is incorrect because there are three bridges connecting O with other islands (J, K, and M).

Please let me know if that helps. Thanks!

Get the most out of your LSAT Prep Plus subscription.

Analyze and track your performance with our Testing and Analytics Package.